Calculus: Early Transcendentals 8th Edition

Published by Cengage Learning
ISBN 10: 1285741552
ISBN 13: 978-1-28574-155-0

Chapter 7 - Section 7.8 - Improper Integrals - 7.8 Exercises - Page 534: 30

Answer

Diverges.

Work Step by Step

The integrand has a discontinuity at $-1$, so this is a type 2.b improper integral. $\displaystyle \int_{a}^{b}f(x)dx=\lim_{t\rightarrow a^{+}}\int_{t}^{b}f(x)dx$ $\displaystyle \int_{-1}^{2}\frac{x}{(x+1)^{2}}dx=\lim_{t\rightarrow-1^{+}}\int_{t}^{2}\frac{x}{(x+1)^{2}}dx$ $\left[\begin{array}{ll} u=x+1 & \\ du=dx & \\ x=-1\Rightarrow u=0, & x=2\Rightarrow u=3 \end{array}\right]$ $=\displaystyle \lim_{u\rightarrow 0^{+}}\int_{u}^{3}\frac{u-1}{u^{2}}dx$ $=\displaystyle \lim_{u\rightarrow 0^{+}}\left[\ln|u|-\frac{u^{-1}}{-1}\right]_{u}^{3}$ $=\displaystyle \lim_{u\rightarrow 0^{+}}\left[\ln|3|+\frac{1}{3}\right]-\lim_{u\rightarrow 0^{+}}\left[\ln|u|+\frac{1}{u}\right]$ Diverges, because $\displaystyle \lim_{u\rightarrow 0^{+}}\left[\ln|u|+\frac{1}{u}\right]=\lim_{u\rightarrow 0^{+}}\left[(u\ln u+1)\frac{1}{u}\right]$ Now, $\displaystyle \lim_{u\rightarrow 0^{+}}u\ln u =\displaystyle \lim_{u\rightarrow 0^{+}}\frac{\ln u}{\frac{1}{u}}=$(L,Hosp)$=\displaystyle \lim_{u\rightarrow 0^{+}}\frac{1}{u(-u^{-2})}=\lim_{u\rightarrow 0^{+}}(-u)=0$, so $\displaystyle \lim_{u\rightarrow 0^{+}}\left[\ln|u|+\frac{1}{u}\right]=(0+1)\lim_{u\rightarrow 0^{+}}\frac{1}{u}=\infty$
Update this answer!

You can help us out by revising, improving and updating this answer.

Update this answer

After you claim an answer you’ll have 24 hours to send in a draft. An editor will review the submission and either publish your submission or provide feedback.